LSAT and Law School Admissions Forum

Get expert LSAT preparation and law school admissions advice from PowerScore Test Preparation.

 Johnclem
  • Posts: 122
  • Joined: Dec 31, 2015
|
#28214
Hello ,
I can see why B is the answer . But what's wrong with A? To me it seems to weaken the conclusion when negated .


1- the courts determined that information writing for a machine does not fit into either copyright of the patent catergory .
C: therefore the profit rights of computer software writers remain unprotected in Jawade.

A) negated says : computer software writes are an influential enough group for the govenement to consider modifying existing copyright laws in order to protect this groups profit to rights .

Thanks
John
 David Boyle
PowerScore Staff
  • PowerScore Staff
  • Posts: 836
  • Joined: Jun 07, 2013
|
#28249
Johnclem wrote:Hello ,
I can see why B is the answer . But what's wrong with A? To me it seems to weaken the conclusion when negated .


1- the courts determined that information writing for a machine does not fit into either copyright of the patent catergory .
C: therefore the profit rights of computer software writers remain unprotected in Jawade.

A) negated says : computer software writes are an influential enough group for the govenement to consider modifying existing copyright laws in order to protect this groups profit to rights .

Thanks
John

Hello John,

I assume you are speaking of question #6 in that section. --Answer A may be a strengthener of the idea that software writers are unprotected, but that doesn't mean it's an assumption. (The negated version of answer A says that they might be protected, so that would weaken the idea that software writers are a weak group.)

Hope this helps,
David
 MikeJones
  • Posts: 31
  • Joined: Oct 02, 2017
|
#40241
Johnclem wrote:Hello ,
I can see why B is the answer . But what's wrong with A? To me it seems to weaken the conclusion when negated .


1- the courts determined that information writing for a machine does not fit into either copyright of the patent catergory .
C: therefore the profit rights of computer software writers remain unprotected in Jawade.

A) negated says : computer software writes are an influential enough group for the govenement to consider modifying existing copyright laws in order to protect this groups profit to rights .

Thanks
John
Answer A isn't relevant to the conclusion because the conclusion is discussing a current state of affairs. Answer A is addressing something that might be possible in the future, which ultimately has to bearing on what "remains unprotected," meaning the present.
 Francis O'Rourke
PowerScore Staff
  • PowerScore Staff
  • Posts: 471
  • Joined: Mar 10, 2017
|
#40290
Hi Mike,

Are you explaining why answer choice (A) would not strengthen the conclusion? If so, I would not go quite as far as you do in focusing on the temporal aspect of the answer choice. It may be that in Jawade, the government can retroactively apply copyrights. Answer choice (A) says that this is impossible because software writers are not influential enough to garner government support. In this way, it does strengthen the argument by discounting one possibility of how the software writers may win their profit rights.

In any case, answer choice (A) fails as a necessary assumption for this argument, since we have no way of knowing what the speaker must believe about the influence of the software writers on the government.
 haganskl
  • Posts: 43
  • Joined: May 30, 2019
|
#75472
Hello!
Would (E) be correct if the very end had been replaced with “since computer software writers demanded that their rights be protected.” Or is this still not enough?
Maybe if it had been replaced with “have not been modified in a manner which allows the information written for a machine to it into either category.”

Thanks in advance.

P.S. I understand why B is correct. It was my prephrase. Smh
However, E made sense to me too. After review I realized my error. The number of times the laws have been written or rewritten is irrelevant. The content is what matters. Right?
 Adam Tyson
PowerScore Staff
  • PowerScore Staff
  • Posts: 5400
  • Joined: Apr 14, 2011
|
#75959
Interesting question, haganskl! If answer E had said "Copyright laws and patent laws in Jawade have not been modified since the court made their determination," it would be much more attractive, but I think it would have the same problem as answer A. The issue isn't about what happened AFTER the decision, but what was the situation AT THE TIME the decision was made. Once the court said sorry, you don't get either patent or copyright protection, were computer software writers in that moment completely unprotected, of were there other laws in place at the time that covered them? The author must have assumed that in that moment, they had no such protection - they were totally unprotected. He didn't have to assume anything about what might have happened afterwards, either through new court decisions or new legislation or anything else.

Focus on the timeframe, which is that they "remained unprotected." The author is assuming that they were unprotected before that court decision came down!
 haganskl
  • Posts: 43
  • Joined: May 30, 2019
|
#76177
Thank you!

Get the most out of your LSAT Prep Plus subscription.

Analyze and track your performance with our Testing and Analytics Package.